JEE Main & Advanced Physics Electrostatics & Capacitance Question Bank Electric Field and Potential

  • question_answer
    Figure shows three points A, B and C in a region of uniform electric field \[\overrightarrow{E}\]. The line AB is perpendicular and BC is parallel to the field lines. Then which of the following holds good. Where \[{{V}_{A}},{{V}_{B}}\] and \[{{V}_{C}}\] represent the electric potential at points A, B and C respectively [CPMT 2004; MP PMT 2005]

    A)            \[{{V}_{A}}={{V}_{B}}={{V}_{C}}\]

    B)            \[{{V}_{A}}={{V}_{B}}>{{V}_{C}}\]

    C)                    \[{{V}_{A}}={{V}_{B}}<{{V}_{C}}\]

    D)                    \[{{V}_{A}}>{{V}_{B}}={{V}_{C}}\]

    Correct Answer: B

    Solution :

                 In the direction of electric field potential decreases.


You need to login to perform this action.
You will be redirected in 3 sec spinner